The rat population in major metropolitan city is given by the formula n(t)=40e^0.015t where t is measured in years since 1991 and n(t) is measured in millions. What does the model predict the rat population was in the year 2008?

Answers

Answer 1

To use the model we need to find the value of t. To do this we substract the year we want to know from the year the model began, then:

[tex]t=2008-1991=17[/tex]

Now that we have t we plug it in the function:

[tex]n(17)=40e^{0.015\cdot17}=51.618[/tex]

Therefore the model predict that there were 51.618 millions of rats in 2008.


Related Questions

What is the value of f(3) on the following graph?

Answers

Answer

f(3) = -2

Explanation

We are asked to find the value of f(3) from the graph.

This means we are looking for the value of f(x) or y on the graph, at a point where x = 3.

From the graph, we can see that at the point where x = 3, y = -2

Hence, f(3) = -2

Hope this Helps!!!

Determine if the ordered pair provided is a solution to the linear system:3x+7y=1 and 2x+4y=0; (2,3) The system has no solution as the lines are parallel. The ordered pair (2, 3) is not a solution to the system. Yes, (2, 3) is a solution to the system. The system has no solution as the lines are perpendicular.

Answers

Answer:

The correct answer is:

The ordered pair (2, 3) is not a solution to the system.

Explanation:

The system given is:

[tex]\begin{cases}3x+7y={1} \\ 2x+4y={0}\end{cases}[/tex]

If (2, 3) is a solution of the system, then replacing x = 2 and y = 3 on both equations should give a correct result and the same on both equatiions.

In the first equation;

[tex]\begin{gathered} 3\cdot2+7\cdot3=1 \\ 6+21=1 \\ 27=1 \end{gathered}[/tex]

We can see that this result is not true, as 27 is not equal to 1.

In the second equation:

[tex]\begin{gathered} 2\cdot2+4\cdot3=0 \\ 4+12=0 \\ 16=0 \end{gathered}[/tex]

Once again, a false result.

To see in the system has equations, let's solve for x in the second equation:

[tex]\begin{gathered} 2x+4y=0 \\ 2x=-4y \\ x=-2y \end{gathered}[/tex]

Now, we can use substitution in the first equation:

[tex]3(-2y)+7y=1[/tex]

And solve for y:

[tex]\begin{gathered} -6y+7y=1 \\ y=1 \end{gathered}[/tex]

Now, we can find the value of x:

[tex]x=-2\cdot1=-2[/tex]

The solution to the system is (-2, 1)

Thus, the correct option is "The ordered pair (2, 3) is not a solution to the system"

Bill Jensen deposits $8500 with Bank of America in an investment paying 5% compounded semiannually. Find the interest in 6 years

Answers

Amount deposited = $8500

Rate = 5%

time for interest = 6years

Compounded semiannually

The formula for semiannually is

[tex]A=P(1+\frac{r}{100n})^{nt}[/tex]

From the given information

P = $8500

r = 5

t = 6

Since the investment was compounded semiannually then

n = 2

Substitute the values into the formula

This gives

[tex]A=8500(1+\frac{5}{100\times2})^{6\times2}[/tex]

Solve for A

[tex]\begin{gathered} A=8500(1+0.025)^{12} \\ A=8500(1.025)^{12} \\ A=11431.56 \end{gathered}[/tex]

To find the interest

Recall

[tex]I=A-P[/tex]

Where I, is the interest

Hence

[tex]\begin{gathered} I=\text{\$}11431.56-\text{\$}8500 \\ I=\text{\$}2931.56 \end{gathered}[/tex]

I think of a number.
I add 5 to it and then double the result.
I then subtract 10 from this answer.
I then subtract the original number I thought of.
Using algebra and a pronumeral to represent the number I think of, explain
why I get back to the number I started with.

Answers

Answer: [2(x + 5)] - 10 - x = 2x+10-10-x = 2x-x = x

Step-by-step explanation:

I think of a number, represented by the variable/pronumeral x.

I add 5 to it: x + 5

then double the result: 2(x + 5)

I then subtract 10 from this answer: [2(x + 5)] - 10

I then subtract the original number I thought of: [2(x + 5)] - 10 - x

Simplifying the expression will explain why you get the original number.

[2(x + 5)] - 10 - x = 2x+10-10-x = 2x-x = x.  

Given the zeros of the following polynomial 2 +2i, 3, - 4 select the corresponding factors AND the polynomia O (x + 2i) (2 - 2i) (2 - 3)(x+4) o f(c) = 24 - 23 822 - 42 - 48 0 (2 – 2i) (x + 2i) (2+3)(– 4) 24 – 13 + 82 40 - 48 0 (0 - 2) (+2)(x - 3)(x +4) 24 - 23 - 822 + 4x + 48 1 3 N

Answers

a)

d)

1) Since the zeros of that polynomial were given, then we can write it into the factored form. Note that there are 4 zeros, so we can write:

[tex]\begin{gathered} (x-x_1)(x-x_2)(x-x_3)(x-x_4)=0 \\ (x-(-2i))(x-2i)(x-3)(x-(-4))=0 \\ (x+2i))(x-2i)(x-3)(x+4))=0 \end{gathered}[/tex]

2) To find out the corresponding polynomial then we can expand it by rewriting "i" as -1

[tex]\begin{gathered} (x+2i))(x-2i)(x-3)(x+4) \\ (x+2i)(x-2i)=x^2+4 \\ (x-3)(x+4)=x^2+4x-3x-12 \\ (x^2+4)(x^2+x-12) \\ x^4+x^3-8x^2+4x-48 \end{gathered}[/tex]

3) Hence, the answers are

a)

d)

[tex]x^4+x^3-8x^2+4x-48[/tex]

A batting cage charges a flat fee of $5 to practice and th Write an equation that models the charges (C) in terms of the number of bucket balls (b) that you use: O C = 1.50 b + 5 O C = 5 b + 1.50 6 Ob = 1.60 C + 5 Ob = 5 C + 1.50

Answers

we have

C -----> total charge

b -----> number of buckets of balls

Remmeber that

the equation of the line in slope intercept form is equal to

y=mx+b

where

m is the slope and b is the initial value or y-intercept

In this problem

m=$1.50 per buckey

b=$5

therefore

y=1.50x+5

or

C=1.50b+5

answer is first option

Use a calculator to find the values of X. Round sides to the nearest 10th and angles to the nearest whole number. Use sin or COS as appropriate.

Answers

Given the information about the triangle, we can use the cosine function on angle x to get the following:

[tex]\begin{gathered} \cos x=\frac{\text{adjacent side}}{hypotenuse}=\frac{7}{16} \\ \Rightarrow\cos x=\frac{7}{16} \end{gathered}[/tex]

solving for x, we get:

[tex]\begin{gathered} \cos x=\frac{7}{16} \\ \Rightarrow x=\cos ^{-1}(\frac{7}{16})=64.1 \\ x=61.1\degree \end{gathered}[/tex]

therefore, the value of x is 61.1

what is the constant of proportionality in this proportional relationship? x 2 2-1/2 3 3-1/2 y 5/2 25/8 15/4 35/8. answer choices 4/5, 5/4, 4, 5

Answers

a proportional relationship has the following form:

yyy=

Rami practices his saxophone for 5/6 hour on 4 days each week.
How many hours does Rami practice his saxophone each week?

[] 2/[] Hr

Answers

Answer:

you take 5/6 and multiply it by 4/1.

which gives you 20/6

then reduce it by dividing the top number by the bottom number

 which gives you 3 with a remainder of 2

you then place the remainder over the

This tells you he practicedfor 3 2/6

Step-by-step explanation:

Camera has Alyssa price of $768.95 before tax the sales tax rate is 8.25% final total find the total cost of the camera with sales tax included round your answer to the nearest cent as necessary

Answers

We know that the listed price of the camera is $768.95 and the tax rate is 8.25%.

To find the total cost we must use the next formula

[tex]\text{Total cost }=\text{listed price before tax+(listed price before tax }\cdot\text{rate tax)}[/tex]

Now, we must replace the values in the formula using that 8.25% = 0.0825

[tex]\text{Total cost}=768.95+(768.95\cdot0.0825)[/tex]

Simplifying,

[tex]\text{Total cost}=832.39[/tex]

ANSWER:

$O32

(Simplify your answer, including any radicals. Use integers or fractions for any numbers in the expression.) And determine the quadrants of A+B and A-B.

Answers

Given that:

[tex]\cos A=\frac{5}{13}[/tex]

Where:

[tex]0And:[tex]\cos B=\frac{3}{5}[/tex]

Where:

[tex]0You need to remember that, by definition:[tex]\theta=\cos ^{-1}(\frac{adjacent}{hypotenuse})[/tex]

Therefore, applying this formula, you can find the measure of angles A and B:

[tex]A=\cos ^{-1}(\frac{5}{13})\approx67.38\text{\degree}[/tex][tex]B=\cos ^{-1}(\frac{3}{5})\approx53.13\text{\degree}[/tex]

(a) By definition:

[tex]\sin \mleft(A+B\mright)=sinAcosB+cosAsinB[/tex]

Knowing that:

[tex]\sin \theta=\frac{opposite}{hypotenuse}[/tex]

You can substitute the known values into the equation in order to find the opposite side for angle A:

[tex]\begin{gathered} \sin (67.38\text{\degree)}=\frac{opposite}{13} \\ \\ 13\cdot\sin (67.38\text{\degree)}=opposite \\ \\ opposite\approx12 \end{gathered}[/tex]

Now you know that:

[tex]\sin A=\frac{12}{13}[/tex]

Using the same reasoning for angle B, you get:

[tex]\begin{gathered} \sin (53.13\text{\degree)}=\frac{opposite}{5} \\ \\ 5\cdot\sin (53.13\text{\degree)}=opposite \\ \\ opposite\approx4 \end{gathered}[/tex]

Now you know that:

[tex]\sin B=\frac{4}{5}[/tex]

Substitute values into the Trigonometric Identity:

[tex]\begin{gathered} \sin (A+B)=sinAcosB+cosAsinB \\ \\ \sin (A+B)=(\frac{12}{13})(\frac{3}{5})+(\frac{5}{13})(\frac{4}{5}) \end{gathered}[/tex]

Simplifying, you get:

[tex]\begin{gathered} \sin (A+B)=\frac{36}{65}+\frac{20}{65} \\ \\ \sin (A+B)=\frac{36+20}{65} \end{gathered}[/tex][tex]\sin (A+B)=\frac{56}{65}[/tex]

(b) By definition:

[tex]\sin \mleft(A-B\mright)=sinAcosB-cosAsinB[/tex]

Knowing all the values, you get:

[tex]\begin{gathered} \sin (A-B)=(\frac{12}{13})(\frac{3}{5})-(\frac{5}{13})(\frac{4}{5}) \\ \\ \sin (A-B)=\frac{36-20}{65} \\ \\ \sin (A-B)=\frac{16}{65} \end{gathered}[/tex]

(c) By definition:

[tex]\tan (A+B)=\frac{\tan A+\tan B}{1-\tan A\cdot\tan B}[/tex]

By definition:

[tex]\tan \theta=\frac{opposite}{adjacent}[/tex]

Therefore, in this case:

- For angle A:

[tex]\tan A=\frac{12}{5}[/tex]

- And for angle B:

[tex]\tan B=\frac{4}{3}[/tex]

Therefore, you can substitute values into the formula and simplify:

[tex]\tan (A+B)=\frac{\frac{12}{5}+\frac{4}{3}}{1-(\frac{12}{5}\cdot\frac{4}{3})}[/tex][tex]\tan (A+B)=\frac{\frac{56}{15}}{1-\frac{48}{15}}[/tex][tex]\tan (A+B)=\frac{\frac{56}{15}}{-\frac{11}{5}}[/tex][tex]\tan (A+B)=-\frac{56}{33}[/tex]

(d) By definition:

[tex]\tan (A-B)=\frac{\tan A-\tan B}{1+\tan A\cdot\tan B}[/tex]

Knowing all the values, you can substitute and simplify:

[tex]\tan (A-B)=\frac{\frac{12}{5}-\frac{4}{3}}{1+(\frac{12}{5}\cdot\frac{4}{3})}[/tex][tex]\tan (A-B)=\frac{\frac{16}{15}}{\frac{21}{5}}[/tex][tex]\tan (A-B)=\frac{16}{63}[/tex]

(e) Knowing that:

[tex]\sin (A+B)=\frac{56}{65}[/tex][tex]\tan (A+B)=-\frac{56}{33}[/tex]

Remember the Quadrants:

By definition, in Quadrant II the Sine is positive and the Tangent is negative.

Since in this case, you found that the Sine is positive and the Tangent negative, you can determine that this angle is in the Quadrant II:

[tex]A+B[/tex]

I need help with this practice problem solving This is the subject trigonometry

Answers

Given the fucntion:

f(x) = tanx

Let's graph the function and input the correct values in the box.

• To find the y-intercept of the function, input 0 for x and solve:

[tex]\begin{gathered} f(0)=\tan 0 \\ \\ f(0)=0 \end{gathered}[/tex]

Therefore, the y-intercept is:

(0, 0)

• The period of the function:

The fundamental period of a tangent function is π.

Now, let's find points on the graph:

Therefore, the points are:

[tex]\mleft(-\frac{\pi}{3},-\sqrt{3}\mright),\mleft(-\frac{\pi}{4},-1\mright),\mleft(0,0\mright),\mleft(\frac{\pi}{4},1\mright),\mleft(\frac{\pi}{3},\sqrt{3}\mright)[/tex]

ANSWER:

The tangent function's period is π . The y-intercept of the function is (0, 0).

The points are:

[tex](-\frac{\pi}{3},-\sqrt[]{3}),(-\frac{\pi}{4},-1),(0,0),(\frac{\pi}{4},1),(\frac{\pi}{3},\sqrt[]{3})[/tex]

17. A moving company charges a flat rate of $85 plus and additional $0.17 per mile driven. How far must the company drive to earn at least $100? Round to thenearest mile.x2 84x2 78x2 80x2 88

Answers

ANSWER

88

EXPLANATION

Let x be the miles driven and y be the earnings of the company when they drive for x miles.

If the company charges $0.17 per mile driven plus a flat rate of $85, then the total cost for moving x miles away is,

[tex]y=85+0.17x[/tex]

Now, we have to find for how many miles, x, the company must drive to earn $100 or more,

[tex]85+0.17x\ge100[/tex]

Subtract 85 from both sides,

[tex]\begin{gathered} 85-85+0.17x\geq100-85 \\ \\ 0.17x\ge15 \end{gathered}[/tex]

And divide both sides by 0.17,

[tex]\begin{gathered} \frac{0.17x}{0.17}\ge\frac{15}{0.17} \\ \\ x\ge88.24 \end{gathered}[/tex]

Hence, the company must drive for at least 88 miles to earn at least $100, rounded to the nearest mile.

While munching on some skittles, Bobby the Vampire lost a tooth that just so happened to be one of his fangs. He measured it to be 27 centimeters long. How long was his tooth in inches?​

Answers

Answer: 10.6299

Step-by-step explanation:

There are 0.3937 inches in a cm., So, the length of the tooth in inches is [tex]27(0.3937)=10.6299 \text{ in }[/tex]

Katie opened a savings account and deposited 1,000.00 as principal the account earns 4% interest compounded quarterly what is the balance after 6 years

Answers

P = $1000

r = 4% = 4/100 = 0.04

t = 6 years

Therefore,

[tex]\begin{gathered} A=P(1+\frac{r}{n})^{nt} \\ A=1000(1+\frac{0.04}{4})^{4\times6} \\ A=1000\times1.26973464853 \\ A=1269.73464853 \\ A=\text{ \$1269.73} \end{gathered}[/tex]

(A) The lines have different slopes and intersect at one point?(B) The lines have the same slope and y intercept.?(C) The lines are parallel and do not intersect.?(D) The lines have the same slope and y-intercept.?(E) Infinitely many solutions.?(F) They are the same line.? (G) No Solution ? (H) One solution.?

Answers

Recall that if two lines have the same slop then these two lines are parallel to each other.

the y-intercept is an x-coordinate of the point where the line intersects at the y-axis.

Consider graph 1.

The line intersects at one point and has different slopes, hence this has one solution.

(A) and (H) is true for graph 1.

Consider graph 2.

The lines have the same slope, therefore parallel but there is no y-intercept point.

This have infinitely many solutions.

They are also the same line.

(E) and (F) is true for this graph 2.

Consider graph 3.

The lines have the same slope and they are parallel.

It gives B) is correct

They do not intersect since parallel does not intersect each other.

It gives C) is correct

There is no solution since they do not intersect.

It gives G) is correct.

These lines have intercepted at -1 and -4.

It gives D) is correct

B), D), C), G), D) are correct for graph 3.

Results:

Options Graph

A) 1

B) 3

C) 3

D) 3

E) 2

F) 2

G) 3

H) 1

Find the maximum value:13, 18, 27, 12, 38, 41, 32, 15, 32

Answers

We can find the maximum value by creating a list of the provided numbers from the smallest to the largest.

[tex]12,13,15,18,27,32,32,38,41[/tex]

As we see on the list, the last number and the largest is 41. Some tools are used to solve this kind of problem like the diagram of leaves and stems, a table os fre

The scale factor on a floor plan is 1 in8 ft. What is the actual distance represented by a 2.5 inches on the floor plan

Answers

Given:

Scale factor = 1 inch 8ft

Floor Plan measurement = 2.5 inches

Solution

We should re-write the scale factor in units of inches only.

Recall that:

[tex]1\text{ f}eet\text{ = 12 inches}[/tex]

Then, the scale-factor in inch:

[tex]\begin{gathered} \text{Scale factor = 1 + 8 }\times\text{ 12} \\ =\text{ 1 + 96 } \\ =\text{ 97 inches} \end{gathered}[/tex]

We can then find the actual distance by multiplying the represented distance (2.5 inches) by the scale factor.

So, we have:

[tex]\begin{gathered} \text{Actual distance = Represented distance }\times\text{ scale factor} \\ =2.5\text{ }\times\text{ 97} \\ =\text{ }242.5\text{ inches} \end{gathered}[/tex]

Answer: Actual distance = 242.5 inches

I need help with my statistics homework " -compute the range ,sample variance,and sample standard deviation cost."

Answers

We need to find the range, sample variance, and sample standard deviation cost.

The range is already given: $247. It can be found by subtracting the least from the greatest value:

[tex]466-219=247[/tex]

Now, in order to find the sample variance and the sample standard deviation, we first need to find the mean of the sample:

[tex]\text{ mean }=\text{ }\frac{415+466+400+219}{4}=\frac{1500}{4}=375[/tex]

Now, we can find the sample variance s² using the formula:

[tex]s²=\frac{\sum_{i\mathop{=}1}^n(x_i-\text{ mean})²}{n-1}[/tex]

where n is the number of values (n = 4) and the xi are the values of the sample.

We obtain:

[tex]\begin{gathered} s²=\frac{(415-375)²+(466-375)²+(400-375)²+(219-375)²}{4-1} \\ \\ s²=\frac{40²+91²+25²+(-156)²}{3} \\ \\ s²=\frac{1600+8281+625+24336}{3} \\ \\ s²=\frac{34842}{3} \\ \\ s²=11614 \end{gathered}[/tex]

Now, the sample standard deviation s is the square root of the sample variance:

[tex]\begin{gathered} s=\sqrt{11614} \\ \\ s\cong107.8 \\ \\ s\cong108 \end{gathered}[/tex]

Therefore, rounding to the nearest whole numbers, the answers are:

Answer

range: $247

s² = 11614 dollars²

s ≅ $108

Find the exact value of sin A and cos A where a = 9 and b = 10 and

Answers

Given data:

a=9 , b = 10

use the phythagoras theorem,

[tex]c=\sqrt[]{a^2+b}^2[/tex][tex]\begin{gathered} c=\sqrt[]{9^2+10^2} \\ c=\sqrt[]{81+100} \\ =\sqrt[]{181} \end{gathered}[/tex]

thus,

[tex]\sin A=\frac{opp}{\text{hypo}}[/tex][tex]\text{sinA}=\frac{9}{\sqrt[]{181}}[/tex]

and,

[tex]undefined[/tex]

Solve the right triangle. Write your answers in simplified, rationalized form. DO NOT ROUND!

Answers

base = FG = root 30

perpendicular HG = x

angle = 45 degrees,

we know that

[tex]\text{tan}\theta=\frac{perpendicualr}{base}[/tex][tex]\tan 45=\frac{HG}{\sqrt[]{3}}[/tex][tex]\begin{gathered} 1=\frac{HG}{\sqrt[]{3}} \\ HG=\sqrt[]{3} \end{gathered}[/tex]

so, the value of HG = root 3

30 randomly selected students took the statistics final. If the sample mean was 84, and the standard deviation was 12.2, construct a 99% confidence interval for the mean score of all students

Answers

The confidence interval for the mean score of the 30 randomly selected students is: 99% CI {78.26, 89.73}

What is confidence interval?

Confidence interval is the range of values for which which is expected to have the values at a certain percentage of the times.

How to construct a 99% confidence interval

Given data form the question

99% confidence interval

30 randomly selected students

mean sample = 84

Standard deviation = 12.2

Definition of variables

confidence level, CI = 99%

mean sample, X = 84

standard deviation, SD = 12.2

Z score, z = 2.576

from z table z score of 99%confidence interval = 2.576

sample size, n = 30

The formula for the confidence interval is given by

[tex]CI=X+Z\frac{SD}{\sqrt{n} }[/tex]    OR    [tex]CI=X-Z\frac{SD}{\sqrt{n} }[/tex]  

[tex]=84+2.576\frac{12.2}{\sqrt{30} }[/tex]

=[tex]=84+2.576*2.2274[/tex]

= 84 + 5.7378     OR       84 - 5.7378

= 89.7378           OR        78.2622

=  89.73 to 78.26  

The confidence interval for the mean score of all students is 78.26 to 89.78

Learn more about confidence interval at: https://brainly.com/question/17097944

#SPJ1

I need help on 3 it says find the value of x round each answer to the nearest tenth

Answers

In problem 3, we have a right triangle with:

• cathetus ,a = 7,,

,

• cathetus ,b = x,,

,

• and hypotenuse ,h = 9,.

Pigatoras Theorem states that:

[tex]h^2=a^2+b^2.[/tex]

Where a and b are cathetus and h the hypotenuse.

Replacing the data of the problem in the equation above, we have:

[tex]9^2=7^2+x^2.[/tex]

Solving for x the last equation, we get:

[tex]\begin{gathered} 81=49+x^2, \\ x^2=81-49, \\ x^2=32, \\ x=\sqrt[]{32}\cong5.7. \end{gathered}[/tex]

Answer

The value of x to the nearest tenth is 5.7.

(2i) - (11+2i) complex numbers

Answers

= -11
Hope this helps :)

At a point on the ground 35 ft from base of a tree, the distance to the top of the tree is 1 ft more than 3 times the height of the tree. Find the height of the tree. The height of the tree is ___. (ft^3, ft^2, or ft)(Simply your answer. Round to the nearest foot as needed)

Answers

At a point on the ground 35 ft from the base of a tree, the distance to the top of the tree is 1 ft more than 3 times the height of the tree. Find the height of the tree

see the attached figure to better understand the problem

Applying the Pythagorean Theorem

(3h+1)^2=h^2+35^2

9h^2+6h+1=h^2+1,225

solve for h

9h^2-h^2+6h+1-1,225=0

8h^2+6h-1,224=0

Solve the quadratic equation

Using a graphing tool

the solution is

h=12 ft

Zaria is making pipe cleaner flowers for
her friends. She has 215 pipe cleaners.
How many flowers can she make with 3
pipe cleaners in each?
[?] flowers and pipe cleaners leftover
I
Answer
Enter

Answers

We can get the answer by dividing 215 by 3

What is dividing?

One of the four fundamental arithmetic operations, or ways to combine numbers to create new ones, is division. The other operations are multiplication, addition, and subtraction. The process of counting the instances in which one integer is included into the others is the most fundamental definition of the division of two natural numbers. This amount need not be an integer. For instance, if twenty apples are divided equally among four people, everyone will get five of them.

We can get the answer by dividing 215 by 3

215/3 = 71.67

Hence, 71 flowers are made

To know more about dividing, click on the link

https://brainly.com/question/1622425

#SPJ9

what is the final cost of the purchase at discount heaven?

Answers

First, let's sum the single costs of each item.

[tex]32+32+20=84[/tex]

Because they are buying 1 jacket, 2 pairs of jeans, and 1 vest. So, the subtotal of these items is $84. (At Discount Heaven)

Then, we apply a 7% sales tax.

[tex]84+0.07\cdot84=84+5.88=89.88[/tex]

As you can observe, the sales tax is $5.88 for all the items purchased, and the total cost they have to pay is $89.88.

Which operation results in a binomial?+(3y6 + 4)(9y12 - 12y6 + 16)ResetNextntum. All rights reserved.

Answers

Answer:

Explanations:

According to the question, we need to determine which of the signs will fit in that will make the expression a binomial.

In simple terms, a binomial is a two-term algebraic expression that contains variable, coefficient, exponents, and constant.

We need to determine the required sign by using the trial and error method.

Using the positive sign (+) first, we will have:

[tex]\begin{gathered} =\mleft(3y^6+4\mright)+(9y^{12}-12y^6+16) \\ =3y^6+4+9y^{12}-12y^6+16 \\ =3y^6-12y^6+4+9y^{12}+16 \\ =-9y^6+9y^{12}+20 \end{gathered}[/tex]

Using the product sign, this will be expressed as:

[tex]\begin{gathered} (3y^6+4)\cdot(9y^{12}-12y^6+16) \\ (3y^6+4)\cdot\lbrack(3y^6)^2-(3y^6)(4)^{}+4^2)\rbrack \end{gathered}[/tex]

According to the sum of two cubes;

[tex]a^3+b^3=\mleft(a+b\mright)•(a^2-ab+b^2)[/tex]

Comparing this with the expression above, we will see that a = 3y^6 and

b = 4. This means that the resulting expression above can be written as a sum of two cubes to have;

[tex]\begin{gathered} (3y^6+4)\cdot\lbrack(3y^6)^2-(3y^6)(4)^{}+4^2)\rbrack^{} \\ =(3y^6)^3-4(3y^6)^2+4(3y^6)^2+16(3y^6)+4(3y^6)^2-16(3y^6)+4^3 \\ \end{gathered}[/tex]

Collect the like terms:

[tex]undefined[/tex]

The following probability table shows probabilities concerning Favorite Subject and Gender. What is the probability of selecting an individual who is a female or prefers science?



Gender Favorite Subject Total
Math English Science
Male 0.200 0.050 0.175 0.425
Female 0.100 0.325 0.150 0.575
Total 0.300 0.375 0.325 1.000

Answers

Answer: 2

Step-by-step explanation: 0.300 0.375 0.325 1.000 = 2

AMNP ~ AQRP N x + 8 28 M 24 P 3x - 9 R Create a proportion and find the length of side PR*

Answers

Using thales theorem:

[tex]\begin{gathered} \frac{24}{28}=\frac{x+8}{3x-9} \\ 24(3x-9)=28(x+8) \\ 72x-216=28x+224 \\ 44x=440 \\ x=\frac{440}{44} \\ x=10 \\ PR=3(10)-9=21 \end{gathered}[/tex]

Other Questions
4) What is perimeter of this shape? * 4 cm 2 cm Question 11You read a news report about a recent large volcanic eruption. It was reportedthat the eruption was highly explosive with viscous lava. What type of volcano waslikely responsible for the eruption?(Hint: the most explosive, dangerous kind)A cinder coneB compositeCshield Solve, graph and write the solution in interval notation: |2x1|>5 Identify the center and the radius of the circle.(x - 1)^2+ (y + 3) = 4 solve by using quadratic formula25c^2 + 40c + 16= 0 What is the value of 10 1 Find the greatest common factor of the following monomials. 28g^5h^2 12g^6h^5 Mosses are located in which zone of deciduous forests? a. tree stratum b. ground c. shrub d. herb please select the best answer from the choices provided a b c d determine whether AB and AC are parallel,perpendicular,or neither.A(9,-3) , B(9,4), C(-2,10), D(-2,6) you add 13.90 ml of 0.370 m ba(oh)2(aq) to 45.00 ml of 0.300 m hydrochloric acid, hcl(aq). if necessary use ka and kb values from the equation sheet provided chem iii eqn sheet a. enter the formula of the chemical species predominantly present in solution that will determine the ph. formula h3o preview: h3o (aq) b. what is the ph? 0.848 Nora has a job where she has a take home salary each month of $2400. if Nora wants to spend no more than 15% of her monthly take home salary on her car payment, how much can she afford? Whats 1+1 I need to know asap Ms. lange drove about 150kms east from la sarre, to senneterre, quebec. she drove another 75kms north to lebel-sur-quevillon, what is the approximate air distance from la sarre to lebel-sur-quevillon What's the midpoint of line segment (5,10) , and (-3,6)? which of the following statements best characterizes delivery reliability? a company that delivers more frequently than its competitors a company that always delivers on the same day of the week a company that delivers faster than its competitors a company that has a computerized delivery scheduling system a company that always delivers at the promised time A ball is thrown from an initial height of 1 meter with an initial upward velocity of 7 m/s. The balls height h (in meters) after t seconds is given by the following. h=1+7t-5t^2Find all values of t for which the balls height is 2 meters.Round the answer(s) to the nearest hundredth Forces with magnitudes of v = 135 newtons and u = 280 newtons act on a hook (see figure). The angle between the two forces is 45. Find the direction and magnitude of the resultant of these forces. (Hint: Write the vector representing each force in component form, then add the vectors. Round your answers to two decimal places.) Hello,Can you help me with question 1: Evaluate the given binomial coefficient Given a family with four children, find the probability of the event. All are boys. The probability that all are boys A metal pole to hang banners and advertisements is attached to a brickbuilding to form a right angle. A diagonal brace is placed 5 feet below thepole to give support. What is the length in feet of the pole?13 ft5 ft